LSAT and Law School Admissions Forum

Get expert LSAT preparation and law school admissions advice from PowerScore Test Preparation.

User avatar
 Dave Killoran
PowerScore Staff
  • PowerScore Staff
  • Posts: 5852
  • Joined: Mar 25, 2011
|
#44134
Setup and Rule Diagram Explanation

This is a Grouping: Partially Defined game.

This is an interesting Grouping game in that movement from one sub-group to another depends on the ability to obtain votes. The game scenario contains a considerable amount of information about the promotion process, including:
  • At least one assistant and one associate are promoted each year.

    An assistant is promoted when a majority of both associates and partners votes for promotion.

    An associate is promoted when a majority partners votes for promotion.

    Every eligible voter (partners and associates) votes in each promotion review.
The rules are fairly simple grouping rules, and can be represented as follows:
D92_Game_#4_setup_diagram 1.png
Based on the scenario information and the rules, several inferences can be made:
  • 1. J cannot be promoted this year because both O and H will not vote for him.

    2. O must be promoted this year because he is the only associate.
With this information, we are ready to attack the questions.
You do not have the required permissions to view the files attached to this post.
 miruke
  • Posts: 8
  • Joined: Jan 23, 2014
|
#14484
Hello,

Im pretty confused about the explanation of question 21.Why is the answer D.I tried to understand how the rationale explained A and C being incorrect,but how can we determine L,S,W will be promoted to associates,based on the not law that O can't vote for G,J,T and the local rule stating R given G one vote.Help please!

Question 22.I kind of understand the 3:2 majority vote rationale for Johnson,but why can't it be one person who needs to be promoted to fulfill the minimum requirement for advancement? Because there will be 3 partners next year and 1 associate,can't the ratio possibly be 3:1 and Johnson possibly be promoted IF the majority votes?!I thought this was a trick question seeing that we have no info about next year's election.

Thanks!
 BethRibet
PowerScore Staff
  • PowerScore Staff
  • Posts: 200
  • Joined: Oct 17, 2012
|
#14485
Hi Miruke,

On question 21, the people who can vote for an assistant to be promoted are Rafael, Harrison and Olivos. To be promoted, the assistant must receive a majority, or 2 votes at least. We know that Rafael has voted to promote Ganz and Wilford, and Harrison to promote Lowry and Stefano, so the remaining question is only, which ones can get an additional vote from Olivos. From the initial rule, we know that Olivos will not vote for Ganz, Johnson or Turner. Johnson and Turner are irrelevant for this question, but this tells us that Ganz will not get a second vote. So the three remaining candidates for promotion are Wilford, Lowry and Stefano, and that corresponds to answer choice D, and to no others (noting that the question asks for a "complete" list of those who could be promoted).

On question 22, we already know that neither Olivos and Harrison will vote for Johnson. That is, to get a majority, he needs three other votes to outweigh those two. Rafael can be one of those three, but there must be two others who will vote for Johnson, in order to get him a three vote majority. Therefore, two more new associates must be available to vote for him. That corresponds to answer choice B, as the question asks what is the "smallest" or bare minimum number of assistants who must have been promoted to associate, in order to make Johnson's promotion possible during the next year's voting.

Hope this helps!
Beth
 miruke
  • Posts: 8
  • Joined: Jan 23, 2014
|
#14486
ok so for question 21.When looking for the possible list,I should focus on only the list of people given to me in the local rule question?I think i got confused because I factored in all the associates.I get it now.

Q22-So,i know language is important,so this questions states for NEXT YEARS review,how do we know that O and H will still not be voting for Johnson?Is that an implication the LSAT authors want us to assume?It gets kind of confusing because one minute the Lsat authors get upset when we assuming then on the next question we are expected to lol.

Thanks!
 Robert Carroll
PowerScore Staff
  • PowerScore Staff
  • Posts: 1787
  • Joined: Dec 06, 2013
|
#14495
Miruke,

For question 21, you need only find those people who can't be on the roster of associates, and check each answer choice to see if it includes one of the people that can't be on that roster. Finding out who can't be on allows you to eliminate wrong answer choices; you're not looking for a list that must be true, but the one list that could be true. All other lists CANNOT be true.

For question 22, note the language in the setup: "Harrison never votes for promoting Johnson or Wilford." Because there is a global condition that (for instance) Harrison "never" does a certain thing, you don't need to assume anything - the game explicitly tells you that this condition applies not just to this promotion review, but all future promotion reviews. Unless a Suspension question suspends that rule, it applies to all possibilities.

Hope this helps!

Robert
 miruke
  • Posts: 8
  • Joined: Jan 23, 2014
|
#14510
I get it now!
Thank you! :)

Get the most out of your LSAT Prep Plus subscription.

Analyze and track your performance with our Testing and Analytics Package.